What dimensions minimize the surface area of a box with a fixed volume?

Click For Summary
To minimize the surface area of a box with a fixed volume, the box's dimensions must be determined based on the relationship between length, width, and height. Given that the length is eight times the width, the volume equation V = lwh leads to a derived surface area formula. By calculating the derivative of the surface area equation and setting it to zero, critical points are found, leading to the optimal dimensions. The final dimensions can be expressed in terms of the fixed volume V. The solution has been verified as correct through an online grading tool.
UMich1344
Messages
26
Reaction score
0
[SOLVED] Minimizing the Surface Area

Homework Statement



A box has a bottom with one edge 8 times as long as the other. If the box has no top and the volume is fixed at V, what dimensions minimize the surface area?

Homework Equations



V = lwh

SA (with no top) = lw + 2lh + 2wh

The Attempt at a Solution



l = x
w = 8x
h = V/(8x^2)

Finding an equation for the surface area.

SA = lw + 2lh + 2wh
SA = x(8x) + 2x(V/(8x^2)) + 2(8x)(V/(8x^2))
SA = 8x^2 + V/(4x) + 2V/x

Finding the derivative of the equation in order to set it equal to zero to find the critical points, so the minimum can be found.

(d SA)/(d x) = 16x - V/(4x^2) - 2V/(x^2)
(d SA)/(d x) = (64x^3 - 9V) / (4x^2)

(d SA)/(d x) = 0
(64x^3 - 9V) / (4x^2) = 0
(64x^3 - 9V) = 0
64x^3 = 9V
x^3 = (9V)/64
x = ((9V)/64)^(1/3)

Plugging the solution into the equations for the dimensions.

l = x = ((9V)/64)^(1/3)
w = 8x = 8((9V)/64)^(1/3)
h = V/(8(((9V)/64)^(1/3))^2)



I am unsure if I did the right steps in order to find the solution.
Also, I am not very confident in the work I did for each step.
 
Physics news on Phys.org
It looks good to me, except you can simplify 64^(1/3) into 4 so that you can reduce your answers and it looks cleaner.
 
Thanks a lot for looking over it. I just put it into the online grader and it is, in fact, correct.
 
I appreciate it.
 
Question: A clock's minute hand has length 4 and its hour hand has length 3. What is the distance between the tips at the moment when it is increasing most rapidly?(Putnam Exam Question) Answer: Making assumption that both the hands moves at constant angular velocities, the answer is ## \sqrt{7} .## But don't you think this assumption is somewhat doubtful and wrong?

Similar threads

  • · Replies 11 ·
Replies
11
Views
6K
Replies
1
Views
1K
  • · Replies 9 ·
Replies
9
Views
2K
  • · Replies 5 ·
Replies
5
Views
2K
  • · Replies 4 ·
Replies
4
Views
2K
  • · Replies 33 ·
2
Replies
33
Views
3K
  • · Replies 3 ·
Replies
3
Views
5K
  • · Replies 3 ·
Replies
3
Views
2K
  • · Replies 2 ·
Replies
2
Views
1K
  • · Replies 3 ·
Replies
3
Views
3K